Đến nội dung

Hình ảnh

Topic BẤT ĐẲNG THỨC ôn thi vào lớp 10 THPT 2017 - 2018

bất đẳng thức am-gm cauchy bunyakovski minskovski schwarz holder thcs

  • Please log in to reply
Chủ đề này có 299 trả lời

#81
Nguyenphuctang

Nguyenphuctang

    Sĩ quan

  • Banned
  • 499 Bài viết

bạn có dùng phần mềm tính toán ko vậy  :D

Hello em! Nếu em thắc mắc thì xin vui lòng vào nhắn tin riêng với bạn ấy hạn chế đăng lên đây làm loãng topic. Mình thấy câu hỏi của bạn chẳng liên quan gì đến vấn đề đang bàn luận ở đây.

Lời giải bài 33:

Không mất tính tổng quát giả sử $c=max\left \{a,b,c  \right \}$ Ta chứng minh bổ đề:

$$ \frac{a^{2}}{a+3} +\frac{b^{2}}{b+3} +\frac{ab}{4} \geq \frac{(a+b)^{2}}{a+b+6} +\frac{(a+b)^{2}}{16} $$

Bổ đề trên khá chặt và hiển nhiên đúng. 

Chứng minh: Bổ đề trên tương đương với:

$$\Leftrightarrow \frac{9(a-b)^{2}}{(a+3)(b+3)(a+b+6)} \geq  \frac{(a-b)^{2}}{16} $$

Bất đẳng thức cuối đúng vì: 

$$ (a+3)(b+3)(a+b+6) \leq \frac{(a+b+6)^{3}}{4} \leq 128 <144 $$

Quay lại bài toán ta cần chứng minh:

$$ \frac{(3-c)^{2}}{9-c} +\frac{(3-c)^{2}}{16} +\frac{c^{2}}{c+3} + \frac{c(3-c)}{4} \geq \frac{3}{2} \Leftrightarrow \frac{3(c-1)^{2}(c-3)^{2}}{16(9-c)(3+c)} \geq 0 $$

Đẳng thức xảy ra khi $a=b=c=1$ hoặc $a=3; b=c=0 $ $\square$ 


Bài viết đã được chỉnh sửa nội dung bởi Nguyenphuctang: 01-05-2017 - 18:40


#82
TrBaoChis

TrBaoChis

    Hạ sĩ

  • Banned
  • 81 Bài viết

 

    Bài 3*/(sưu tầm)

Cho các số thực a, b, c thoả mãn 0a,b,cvà a+b+cChứng minh rằng:

ab(a+1)+bc(b+1)+ca(c+1)2

 

    Bài 3*/(sưu tầm/do mình quên,đây là một bài trong đề thi chuyên của tỉnh nào đó trong mấy năm gần đây)  

Cho các số thực a, b, c thoả : 0abc1.

Tìm giá trị lớn nhất của biểu thức: Q=a2(bc)+b2(cb)+c2(1c)

 

    Bài 3*(đề thi vào chuyên Toán chuyên Phan Bội Châu) 

 Cho các số thực a, b, c thoả mãn a,b≥0 ; c≥1 ; a+b+c=2.

 Tìm giá trị nhỏ nhất của P = (6-a2-b2-c2) (2-abc)

 

câu 1 là đề NA năm ngoái nha bạn  :D

0 $\leq$ a,b $\leq$ 1 $\Rightarrow$ $\sum$ a(a-1)(b-1) $\geq$ 0 
$\Rightarrow$ $\sum$ $a^2b$ + $\sum$ $a$ $\geq$ $\sum$  $a^2$ + $\sum$ $ab$   
$\rightarrow$ $P$ $\geq$ $(a+b+c)^2$ -$(a+b+c)$ $=$ $(a+b+c)(a+b+c-1)$ $\geq $ 2   


#83
tuaneee111

tuaneee111

    Trung sĩ

  • Thành viên
  • 174 Bài viết

a

 

Hello em! Nếu em thắc mắc thì xin vui lòng vào nhắn tin riêng với bạn ấy hạn chế đăng lên đây làm loãng topic. Mình thấy câu hỏi của bạn chẳng liên quan gì đến vấn đề đang bàn luận ở đây.

Lời giải bài 33:

Không mất tính tổng quát giả sử $c=max\left \{a,b,c  \right \}$ Ta chứng minh bổ đề:

$$ \frac{a^{2}}{a+3} +\frac{b^{2}}{b+3} +\frac{ab}{4} \geq \frac{(a+b)^{2}}{a+b+6} +\frac{(a+b)^{2}}{16} $$

Bổ đề trên khá chặt và hiển nhiên đúng. 

Chứng minh: Bổ đề trên tương đương với:

$$\Leftrightarrow \frac{9(a-b)^{2}}{(a+3)(b+3)(a+b+6)} \geq  \frac{(a-b)^{2}}{16} $$

Bất đẳng thức cuối đúng vì: 

$$ (a+3)(b+3)(a+b+6) \leq \frac{(a+b+6)^{3}}{4} \leq 128 <144 $$

Quay lại bài toán ta cần chứng minh:

$$ \frac{(3-c)^{2}}{9-c} +\frac{(3-c)^{2}}{16} +\frac{c^{2}}{c+3} + \frac{c(3-c)}{4} \geq \frac{3}{2} \Leftrightarrow \frac{3(c-1)^{2}(c-3)^{2}}{16(9-c)(3+c)} \geq 0 $$

Đẳng thức xảy ra khi $a=b=c=1$ hoặc $a=3; b=c=0 $ $\square$ 

anh ơi làm sao tìm đc cái bổ đề kia vậy!


$$\boxed{\boxed{I\heartsuit MATHEMATICAL}}$$

Blog của tôi

:luoi: Sức hấp dẫn của toán học mãnh liệt đến nỗi tôi bắt đầu sao nhãng các môn học khác - Sofia Vasilyevna Kovalevskaya :lol:


#84
TrBaoChis

TrBaoChis

    Hạ sĩ

  • Banned
  • 81 Bài viết

a

 

anh ơi làm sao tìm đc cái bổ đề kia vậy!

hình như cái này là kinh nghiệm cho đi lấy lại gì đó của thầy khanh sy :v



#85
Drago

Drago

    Sĩ quan

  • Thành viên
  • 462 Bài viết

Bài Toán 34: [Sưu tầm]

Cho ba số thực a,b,c. Chứng minh rằng:

$(a^2+1)(b^2+1)(c^2+1)\geq (ab+bc+ca-1)^2$


Bài viết đã được chỉnh sửa nội dung bởi Drago: 02-05-2017 - 09:45

$\mathbb{VTL}$


#86
Nguyenphuctang

Nguyenphuctang

    Sĩ quan

  • Banned
  • 499 Bài viết

a

 

anh ơi làm sao tìm đc cái bổ đề kia vậy!

Tùy vào kinh nghiệm mỗi người khi giải thôi bạn. Nói thẳng tuy mình sở hữu đầy đủ bộ tài liệu về phần này của Sỹ ca nhưng mình thấy cái này có phần may mắn và chẳng có cái chuẩn nào cho việc đánh giá. Miễn sao là dồn biến được.  Còn nhiều lời giải khác nếu bạn quan tâm có thể kiếm trên k2pi (Hồi đó có đăng)

hình như cái này là kinh nghiệm cho đi lấy lại gì đó của thầy khanh sy :v

 

Anh Sỹ không phải giáo viên em nhé, anh Sỹ là kiến trúc sư. 

Nhưng box này là THCS nên mình sẽ đăng những bài nhẹ nhàng hơn xíu.

Bài 35: (Tăng) Cho a, b, c > 0 thỏa mãn abc = 1. Chứng minh rằng: $$\frac{1}{a^{3}+b^{3}+c^{3}}+\frac{1}{ab+bc+ca}\ge \frac{6}{(a^{2}+b^{2}+c^{2})^{2}}$$

Nhắc nhở luôn bạn  Drago : Hãy ghi nguồn của bài toán!


Bài viết đã được chỉnh sửa nội dung bởi Nguyenphuctang: 01-05-2017 - 19:03


#87
Hoang Dinh Nhat

Hoang Dinh Nhat

    Sĩ quan

  • Thành viên
  • 402 Bài viết

$\boxed{\textbf{Bài Toán 34}}$

18279960_1846092788972773_897922199_n.pn

 

BĐT Cần chứng minh $\Leftrightarrow (a^2+1)(b^2+1)(c^2+1)-(ab+bc+ca-1)^2\geq 0$

$\Leftrightarrow a^2b^2c^2-2a^2bc+a^2-2ab^2c-2abc^2+2ab+2ac+b^2+2bc+c^2\geq 0\Leftrightarrow (abc-a-b-c)^2\geq 0$

BĐT cuối đúng nên BĐT được chứng minh


Chấp nhận giới hạn của bản thân, nhưng đừng bao giờ bỏ cuộc

 

 

 

 


#88
Hoang Dinh Nhat

Hoang Dinh Nhat

    Sĩ quan

  • Thành viên
  • 402 Bài viết

Bài 36: ​(sưu tầm)

 

 

 

Hình gửi kèm

  • Capture.PNG

Chấp nhận giới hạn của bản thân, nhưng đừng bao giờ bỏ cuộc

 

 

 

 


#89
sharker

sharker

    Sĩ quan

  • Thành viên
  • 301 Bài viết

Bài 36: ​(sưu tầm)

 


${a^5} + {b^5} \ge {a^2}{b^2}(a + b)$
 $\to \sum {\frac{{ab}}{{{a^5} + {b^5} + ab}} \le } \sum {\frac{{ab}}{{{a^2}{b^2}(a + b) + ab}} = \sum {\frac{1}{{ab(a + b) + 1}}}  = \sum {\frac{{abc}}{{ab(a + b) + abc}}} = \sum {\frac{c}{{a + b + c}}} }  = 1$
 

 

Bài viết đã được chỉnh sửa nội dung bởi sharker: 01-05-2017 - 20:10

Anh sẽ vẫn bên em dù bất cứ nơi đâu

Anh sẽ là hạt bụi bay theo gió

Anh sẽ là ngôi sao trên bầu trời phương Bắc

Anh không bao giờ dừng lại ở một nơi nào

Anh sẽ là ngọn gió thổi qua các ngọn cây

Em sẽ mãi mãi đợi anh chứ ??

will you wait for me forever


#90
sharker

sharker

    Sĩ quan

  • Thành viên
  • 301 Bài viết

Bài 37

Cho a,b,c là 3 cạnh của tam giác

CMR 

$({a^2} + {b^2} + {c^2})\left( {\frac{1}{{{a^2}}} + \frac{1}{{{b^2}}} + \frac{1}{{{c^2}}}} \right) \ge 10$

Bài viết đã được chỉnh sửa nội dung bởi sharker: 01-05-2017 - 20:38

Anh sẽ vẫn bên em dù bất cứ nơi đâu

Anh sẽ là hạt bụi bay theo gió

Anh sẽ là ngôi sao trên bầu trời phương Bắc

Anh không bao giờ dừng lại ở một nơi nào

Anh sẽ là ngọn gió thổi qua các ngọn cây

Em sẽ mãi mãi đợi anh chứ ??

will you wait for me forever


#91
viet9a14124869

viet9a14124869

    Trung úy

  • Thành viên
  • 903 Bài viết

 

Bài 37

Cho a,b,c là 3 cạnh của tam giác

CMR 

$({a^2} + {b^2} + {c^2})\left( {\frac{1}{{{a^2}}} + \frac{1}{{{b^2}}} + \frac{1}{{{c^2}}}} \right) \ge 10$

 

Chắc hẳn còn có điều kiện gì khác ,,bởi ta thấy ngay bài toán sai tại a=b=c :D


                                                                    SÓNG BẮT ĐẦU TỪ GIÓ

                                                                    GIÓ BẮT ĐẦU TỪ ĐÂU ?

                                                                    ANH CŨNG KHÔNG BIẾT NỮA 

                                                                    KHI NÀO...? TA YÊU NHAU .


#92
AnhTran2911

AnhTran2911

    Thượng sĩ

  • Thành viên
  • 230 Bài viết

Tùy vào kinh nghiệm mỗi người khi giải thôi bạn. Nói thẳng tuy mình sở hữu đầy đủ bộ tài liệu về phần này của Sỹ ca nhưng mình thấy cái này có phần may mắn và chẳng có cái chuẩn nào cho việc đánh giá. Miễn sao là dồn biến được.  Còn nhiều lời giải khác nếu bạn quan tâm có thể kiếm trên k2pi (Hồi đó có đăng)

Anh Sỹ không phải giáo viên em nhé, anh Sỹ là kiến trúc sư. 

Nhưng box này là THCS nên mình sẽ đăng những bài nhẹ nhàng hơn xíu.

Bài 35: (Tăng) Cho a, b, c > 0 thỏa mãn abc = 1. Chứng minh rằng: $$\frac{1}{a^{3}+b^{3}+c^{3}}+\frac{1}{ab+bc+ca}\ge \frac{6}{(a^{2}+b^{2}+c^{2})^{2}}$$

Nhắc nhở luôn bạn  Drago : Hãy ghi nguồn của bài toán!

Tăng ơi. Hình như cái ni mi đăng lên fb rồi mà.

SD cái bổ đề $(a^2+b^2+c^2)^2\geq9(a^3+b^3+c^3)$ Với giả thiết $abc=1$ ( $a,b,c\ge0$)


Bài viết đã được chỉnh sửa nội dung bởi AnhTran2911: 01-05-2017 - 21:44

        AQ02

                                 


#93
AnhTran2911

AnhTran2911

    Thượng sĩ

  • Thành viên
  • 230 Bài viết

Chắc hẳn còn có điều kiện gì khác ,,bởi ta thấy ngay bài toán sai tại a=b=c :D

Vâng. đúng là phải thêm đk tam giác đã cho tù. Bài này mình nhớ bạn Nhoang1608 làm 1 lần rồi thì phải.

P/s: Cho mình hỏi có ai làm đc bài số 31 của mình chưa ạ ?


Bài viết đã được chỉnh sửa nội dung bởi AnhTran2911: 01-05-2017 - 21:35

        AQ02

                                 


#94
Drago

Drago

    Sĩ quan

  • Thành viên
  • 462 Bài viết

$\boxed{\textbf{Bài Toán 38}}$[Sưu tầm]

 

Cho $x,y,z$ là các số thực dương thoả mãn $x+y+z=xyz$. Chứng minh rằng:

$$\frac{2}{\sqrt{1+x^2}}+\frac{1}{\sqrt{1+y^2}}+\frac{1}{\sqrt{1+z^2}}\leq \frac{9}{4}$$


Bài viết đã được chỉnh sửa nội dung bởi Drago: 02-05-2017 - 08:09

$\mathbb{VTL}$


#95
Nguyenhuyen_AG

Nguyenhuyen_AG

    Trung úy

  • Thành viên nổi bật 2016
  • 945 Bài viết

Bài 33: (Khánh Sỹ)

Cho các số thực không âm $a,b,c$ thỏa mãn $a+b+c=3$. Chứng minh rằng:

$$ \sum_{cyc} \frac{a^{2}}{a+3} + \frac{ab+bc+ca}{4} \geq \frac{3}{2} $$

 

Ta có

\[\sum \frac{a^{2}}{a+3} + \frac{ab+bc+ca}{4} - \frac{3}{2} = \frac{1}{12(a+3)(b+3)(c+3)} \sum a(3a^2+bc)(b-c)^2 \geqslant 0.\]

 

$\boxed{\textbf{ BÀI TOÁN 31 }}$ 

Cho $ a,b,c\ge 0$ . Chứng Minh Rằng:

$3(a^2+b^2+c^2)(a^2b^2+b^2c^2+c^2a^2)\geq(a^2+ab+b^2)(b^2+bc+c^2)(c^2+ca+a^2)$

 

\[\text{Vế trái  - Vế phải} = \frac14\sum \left[(a+b-c)^2+5a^2+5b^2+c^2\right]c^2(a-b)^2 \geqslant 0.\]


Nguyen Van Huyen
Ho Chi Minh City University Of Transport

#96
Mr Cooper

Mr Cooper

    Sĩ quan

  • Thành viên
  • 496 Bài viết

$\boxed{\textbf{Bài Toán 39}}$ Cho $a,b,c$ là các số thực không âm thỏa mãn $a+b+c=5$. Chứng minh rằng:

\[|(a^2-b^2)(b^2-c^2)(c^2-a^2) \le \sqrt{5}\]

 

Anh Drago nên gõ đề ra để tránh die ảnh


Bài viết đã được chỉnh sửa nội dung bởi Mr Cooper: 02-05-2017 - 07:46


#97
Chu Quang Huy

Chu Quang Huy

    Binh nhì

  • Thành viên mới
  • 12 Bài viết

 

câu 1 là đề NA năm ngoái nha bạn  :D

0 $\leq$ a,b $\leq$ 1 $\Rightarrow$ $\sum$ a(a-1)(b-1) $\geq$ 0 
$\Rightarrow$ $\sum$ $a^2b$ + $\sum$ $a$ $\geq$ $\sum$  $a^2$ + $\sum$ $ab$   
$\rightarrow$ $P$ $\geq$ $(a+b+c)^2$ -$(a+b+c)$ $=$ $(a+b+c)(a+b+c-1)$ $\geq $ 2   

 

Vậy bạn làm được câu cuối không??????



#98
TrBaoChis

TrBaoChis

    Hạ sĩ

  • Banned
  • 81 Bài viết

Vậy bạn làm được câu cuối không??????

câu trường Phan mình chưa làm thì đã đc xem giải rồi  :D  :D



#99
Zz Isaac Newton Zz

Zz Isaac Newton Zz

    Sĩ quan

  • Điều hành viên OLYMPIC
  • 392 Bài viết

$\boxed{\textbf{Bài Toán 39}}$ Cho $a,b,c$ là các số thực không âm thỏa mãn $a+b+c=5$. Chứng minh rằng:

\[|(a^2-b^2)(b^2-c^2)(c^2-a^2) \le \sqrt{5}\]

 

Anh Drago nên gõ đề ra để tránh die ảnh

Hình như giả thiết bài này phải là $a+b+c=\sqrt{5}$ chứ ạ...

Đặt $P=\begin{vmatrix} (a^{2}-b^{2})(b^{2}-c^{2})(c^{2}-a^{2}) \end{vmatrix}$

Giả sử $c=min{a, b, c}$ ta chỉ cần xét $b\geq a.$ Đặt $\left\{\begin{matrix} x=\sqrt{a^{2}-c^{2}} & & \\ y=\sqrt{b^{2}-c^{2}} & & \end{matrix}\right.$ $\Leftrightarrow \left\{\begin{matrix} a=\sqrt{x^{2}+c^{2}} & & \\ b=\sqrt{y^{2}+c^{2}} & & \end{matrix}\right.$ 

Khi đó: $a+b+c=\sqrt{5}=\sqrt{x^{2}+c^{2}}+\sqrt{y^{2}+c^{2}}+c\geq x+y.$

Và $P\leq x^{2}y^{2}(y^{2}-x^{2}).$

Mà $4xy+(x-y)^{2}=(x+y)^{2}.$ Từ đó áp dụng bất đẳng thức $AM-GM$ ta có:

$P^{2}\leq (x+y)^{2}.xy.xy.xy.xy.(x-y)^{2}\leq (x+y)^{2}.\left [ \frac{4xy+(x-y)^{2}}{5} \right ]^{5}=\frac{(x+y)^{12}}{3125}\leq 5.$ Từ đây ta có $P\leq \sqrt{5}.$

Dấu bằng xảy ra khi và chỉ khi $(a, b, c)=(\frac{\sqrt{5}-1}{2}, \frac{\sqrt{5}+1}{2}, 0).$ Kết thúc chứng minh.



#100
NHoang1608

NHoang1608

    Sĩ quan

  • Thành viên
  • 375 Bài viết

$\boxed{\textbf{Bài Toán 38}}$[Sưu tầm]

 

Cho $x,y,z$ là các số thực dương thoả mãn $x+y+z=xyz$. Chứng minh rằng:

$$\frac{2}{\sqrt{1+x^2}}+\frac{1}{\sqrt{1+y^2}}+\frac{1}{\sqrt{1+z^2}}\leq \frac{9}{4}$$

Từ giả thiết ta có $\frac{1}{xy}+\frac{1}{yz}+\frac{1}{zx}=1$

Đặt $(\frac{1}{x};\frac{1}{y};\frac{1}{z})=(a;b;c) \Rightarrow ab+bc+ca=1$

Bài toán quy về chứng minh

                      $P=\frac{2a}{\sqrt{a^{2}+1}}+\frac{b}{\sqrt{b^{2}+1}}+\frac{c}{\sqrt{c^{2}+1}} \leq \frac{9}{4}$

                      $\Leftrightarrow P= \frac{2a}{\sqrt{(a+b)(a+c)}}+\frac{b}{\sqrt{(b+c)(b+a)}}+\frac{c}{\sqrt{(c+a)(c+b)}}\leq \frac{9}{4}$

Ta có $P= \frac{2a}{\sqrt{(a+b)(a+c)}}+\frac{2b}{\sqrt{4(b+c)(b+a)}}+\frac{2c}{\sqrt{4(c+b)(c+a)}}$

                    $\leq_{AM-GM} (\frac{a}{a+b}+\frac{a}{c+a})+(\frac{b}{4(b+c)}+\frac{b}{a+b})+(\frac{c}{4(c+b)}+\frac{c}{c+a})= \frac{9}{4}$

Suy ra đpcm. Dấu bằng xảy ra khi $y=z=\frac{\sqrt{15}}{7},x=\sqrt{15}$. 


Bài viết đã được chỉnh sửa nội dung bởi NHoang1608: 02-05-2017 - 11:47

The greatest danger for most of us is not that our aim is too high and we miss it, but that it is too low and we reach it.

----- Michelangelo----






Được gắn nhãn với một hoặc nhiều trong số những từ khóa sau: bất đẳng thức, am-gm, cauchy, bunyakovski, minskovski, schwarz, holder, thcs

0 người đang xem chủ đề

0 thành viên, 0 khách, 0 thành viên ẩn danh